AAMC CBT8 and 8R OFFICIAL Q&A

This forum made possible through the generous support of SDN members, donors, and sponsors. Thank you.

Vihsadas

No summer
Moderator Emeritus
Lifetime Donor
10+ Year Member
5+ Year Member
15+ Year Member
Joined
Oct 17, 2007
Messages
5,474
Reaction score
56
This is the official Q&A thread for AAMC CBT8 and 8R.

Please post ONLY questions pertaining to AAMC CBT8 and 8R.
Out of respect for people who may not have completed the other exams, do not post questions or material from any other AAMC exam.

Please see this thread for the rules of order before you post.

Good luck on your MCAT!

Members don't see this ad.
 
PS 30 8R

The question says: If the pH of the water sample were high such that all the carbonate is present as CO3^2-, what would be the concentration of Ca^2+? (The Ksp of CaCO3 is 4.8x10^-9.)

Can someone please help with this? Maybe I've been studying too long and can't think straight, but I don't understand what the question is really asking nor what the explanation for this is.

Why would we assume that all of the carbonate has dissociated?

Hmm, I'm not seeing that question in my exam? Can you post the explanation and answer choices?

I believe this is a complex reaction where if the pH is low enough, lots of protons floating around will react with CaCO3. Remember that Carbonate is basic
 
On another note, Q39 required some terrible calculations. Here's the problem: http://www.screencast.com/users/trinhn812/folders/Jing/media/261bfbc3-1051-4e49-ba2e-57af2e92e716

Can someone show me their thought process in the division of (6.57/262.84) and (7.15/286.14). Both equals .025 mol.

For the second calculation I rounded to 7/280 -->1/40= .025 mol. I figured (6.57/262.84) is bigger than .025 and just moved on. Turns out they're both the same! Lucky assumption on my part I guess.
 
It wasn't hard for me to see that both values were close to 1/40. 6.5 * 40 = 260, so 6.57/262.84 isn't too far off. Also, the rest of the answers aren't even close to it.
 
Members don't see this ad :)
coldviva, I am going to try to explain it the way I thought through it. I might wrong, if that is the case someone please correct me. Here it goes:

Since we know that E=hf, and as the light bounces off of the imperfect reflector, some of the energy goes through into another medium the rest of it reflects back. This change in energy of the wave/amplitude would also affect the frequency of the wave as indicated by the E=hf equation.

Hmm.. I'm still a bit confused on this question. http://www.screencast.com/users/trinhn812/folders/Jing/media/507a65a3-f6e9-4da0-a242-8a52e721ae38

EK says that refracted and reflected light will have the same frequency and hence energy as the incident light. However, the sum of the intensity of the refracted and reflected light = the incident light. I'm aware velocity does change unless the medium change, but how does frequency change?
 
Hmm.. I'm still a bit confused on this question. http://www.screencast.com/users/trinhn812/folders/Jing/media/507a65a3-f6e9-4da0-a242-8a52e721ae38

EK says that refracted and reflected light will have the same frequency and hence energy as the incident light. However, the sum of the intensity of the refracted and reflected light = the incident light. I'm aware velocity does change unless the medium change, but how does frequency change?

I thought of it as the doppler effect. So the frequency will change, therefore you get a shift in wavelength as well. If this doesn't make sense I'll try to explain it another way.
 
Hmm.. I'm still a bit confused on this question. http://www.screencast.com/users/trinhn812/folders/Jing/media/507a65a3-f6e9-4da0-a242-8a52e721ae38

EK says that refracted and reflected light will have the same frequency and hence energy as the incident light. However, the sum of the intensity of the refracted and reflected light = the incident light. I'm aware velocity does change unless the medium change, but how does frequency change?

speed is dependent on characteristic of medium, not on frequency. If you are the observer and looking at at as a Doppler effect then ya, it would change since there is relative motion b/w you and the source. (its moving towards you frequencey increases and wavelength decreases.
 
Ouch, this PS hit me on all my weak topics.

2 quick PS questions.

____________________________

PS. #10.

What topic exactly is this questing testing? I was assuming it was asking about unpaired/paired electrons, but the answer given talks about including protons and neutrons.

For a given magnitude of B1, the nucleus with the nonzero precession frequency will be which of the following?

A)

4 He
2

B)

16 O
8

C)

19 F
9

Because protons and neutrons have spin just as electrons do, to guarantee a nonzero net spin, an odd number of nucleons is needed. Of the foils, the only element that has an odd number of nucleons is . Thus, C is the best answer.



D)

208 Pb
82

_______________________________________________

PS, # 49

For this problem there was an accompanying diagram. I was able to reason/guess that q > α given the different values of n; but NO WHERE in the diagram were the variable q ¢ or q. I was/am hoping that my computer didn't display them. Since I had no idea what q ¢ I could only guess between the two.

Was q ¢ suppose to be displayed? If not, how was I suppose to know what the variable was standing for.


A beam of light shines into a transparent medium with parallel surfaces. Part of the beam is reflected back into the air as diagrammed above. (The figure is NOT to scale.) The index of refraction of the medium is 1.5. Which of the following is true?

A)
q < q ¢ and q < &#945;

B)
q = q ¢ and q > &#945;

Because the medium's surfaces are parallel, a perpendicular line drawn to the lower surface of the medium will be parallel to both of the perpendiculars shown in the figure. This means that the angle of incidence at the lower surface will also be a, as will the angle of reflection at the lower surface, and the beam reflecting from the lower surface of the medium will then be a mirror image of the incoming beam, so q¢ = q. Further, because air's index of refraction is about 1.0, Snell's law would show that q > a. Thus, B is the best answer.


C)
q = q ¢ and q < &#945;

D)
q > q ¢ and q > &#945;

Would someone kindly comment on these questions I am also stuck on them.

Thanks in advance
 
bump, please help :confused:

PS-10 was testing how much you remembered about NMR in organic chemistry. MRI=NMR, renamed because non-scientists get scared when you say "nuclear". As you may remember from ochem when they introduced NMR spectroscopy, protons and neutrons have spin too. That's why we do NMR on H-1 and C-13. You can't do NMR on C-12 because it has an even number of nucleons and is therefore unaffected by magnets. Yeah it's kind of weird that an ochem question showed up in PS.

PS-49 has some display issues on some browsers. For "q" read "theta" and for "¢" read "prime". Hopefully this isn't a problem on the real MCAT. If you are still having trouble with this question, it's been answered a few other times on this forum. Write back if you need more help.

And let me just say for the record that the VR section of AAMC8 ought to come with a jar of vaseline. I got totally raped by the "behavior" passage. I got every single question wrong!
 
hey guys, be grateful if anyone could help me out here; starting off with the biological sections:

102) how can u tell that the 2-furaldehyde has been formed under equilibrium conditions and is therefore more stable?? or can i just look at the table and see it's got a higher melting point?
 
hey guys, be grateful if anyone could help me out here; starting off with the biological sections:

102) how can u tell that the 2-furaldehyde has been formed under equilibrium conditions and is therefore more stable?? or can i just look at the table and see it's got a higher melting point?

I think you look at the melting point to see which one is more stable. I'm not 100% sure since I also got this wrong when i took it.:D
 
Members don't see this ad :)
hey guys, be grateful if anyone could help me out here; starting off with the biological sections:

102) how can u tell that the 2-furaldehyde has been formed under equilibrium conditions and is therefore more stable?? or can i just look at the table and see it's got a higher melting point?

The melting point in an of itself is not important. What is important is the temperature under which the reaction took place, and the mix of products that resulted.

Notice that when the conditions were cold, the product had a melting point close to pure cyclohexanone. When the conditions were warm, the product was a mix. When the conditions were hot, the product was almost pure 2-furaldehyde (again, you can tell this by looking at the melting points of the product mixtures).

What does this tell us? Cyclohexanone is the kinetic product. It can form under low temperatures, therefore it has a low activation energy. It forms quickly. But there must be an alternate path leading to 2-furaldehyde, which has a higher activation energy, and therefore requires higher temperatures. It is slower to form. It has a more stable product (because at high temperatures, when there is enough energy to quickly jump either Ea hill, the reaction favors the 2-furaldehyde product). 2-furaldehyde is the thermodynamic product.

Notice (in experiment 2) that even when cyclohexanone is heated, it converts to 2-furaldehyde. This is another clue that 2-furaldehyde is the thermodynamic product. In fact, all of the products (even those in experiment 1A) will eventually become the thermodynamically favored product, the heat just makes it happen faster.
 
The melting point in an of itself is not important. What is important is the temperature under which the reaction took place, and the mix of products that resulted.

Notice that when the conditions were cold, the product had a melting point close to pure cyclohexanone. When the conditions were warm, the product was a mix. When the conditions were hot, the product was almost pure 2-furaldehyde (again, you can tell this by looking at the melting points of the product mixtures).

What does this tell us? Cyclohexanone is the kinetic product. It can form under low temperatures, therefore it has a low activation energy. It forms quickly. But there must be an alternate path leading to 2-furaldehyde, which has a higher activation energy, and therefore requires higher temperatures. It is slower to form. It has a more stable product (because at high temperatures, when there is enough energy to quickly jump either Ea hill, the reaction favors the 2-furaldehyde product). 2-furaldehyde is the thermodynamic product.

Notice (in experiment 2) that even when cyclohexanone is heated, it converts to 2-furaldehyde. This is another clue that 2-furaldehyde is the thermodynamic product. In fact, all of the products (even those in experiment 1A) will eventually become the thermodynamically favored product, the heat just makes it happen faster.

Makes sense, thanks.
 
The melting point in an of itself is not important. What is important is the temperature under which the reaction took place, and the mix of products that resulted.

Notice that when the conditions were cold, the product had a melting point close to pure cyclohexanone. When the conditions were warm, the product was a mix. When the conditions were hot, the product was almost pure 2-furaldehyde (again, you can tell this by looking at the melting points of the product mixtures).

What does this tell us? Cyclohexanone is the kinetic product. It can form under low temperatures, therefore it has a low activation energy. It forms quickly. But there must be an alternate path leading to 2-furaldehyde, which has a higher activation energy, and therefore requires higher temperatures. It is slower to form. It has a more stable product (because at high temperatures, when there is enough energy to quickly jump either Ea hill, the reaction favors the 2-furaldehyde product). 2-furaldehyde is the thermodynamic product.

Notice (in experiment 2) that even when cyclohexanone is heated, it converts to 2-furaldehyde. This is another clue that 2-furaldehyde is the thermodynamic product. In fact, all of the products (even those in experiment 1A) will eventually become the thermodynamically favored product, the heat just makes it happen faster.

thanks a lot, that cleared some things up:)
 
I know question #19 in PS has been explained before (on page 3 of this thread), but I still don't get it, and the explanation is from 1 year ago, so I've decided to ask it again.

The question asks what is the advantage in using a 10m inclined plane over dropping the ball from a height of 10m? The answer choices were:

A) The final velocity of the sphere is smaller.

B) The final velocity of the sphere is larger.

C) The sphere takes longer to reach the bottom.

Explanation: Ignoring air resistance, rotation, and friction, the final velocity of the sphere that drops is the same as the final velocity of the sphere that moves down the plane. Meanwhile, the path is longer on the inclined plane than in free fall, leading to a longer time for the sphere to reach the bottom.

D) The sphere takes shorter to reach the bottom.


I chose A, not C. What I don't get about the answer explanation is that it says that the path is longer for the inclined plane. Why is it longer? The question asks about dropping the ball from 10m height vs. letting it roll down a 10m inclined plane. So isn't the length of the path the same for both? Also, why is A wrong? I assumed that since v = sqrt(2gh), and the g for the inclined plane is really gsintheta, that would mean the v is smaller for the inclined plain. Unless, of course, the inclined plane is longer than 10m, unlike what it says in the question.

Why is C the correct answer? Is it because we use x = 1/2(gsintheta)t^2?

Thanks for your help, this question is driving me nuts!
 
Good catch. The AAMC explanation does not match the question. You are right. For what it's worth, the explanation in post 168 (on my page 4) doesn't make much sense to me.

Here is how I got answer C, based only on the original question. Galileo would have had primitive timing equipment subject to a random error of constant magnitude. By using an inclined plane instead of a straight drop, the time to reach the bottom would be larger, and therefore the random errors of his timing equipment would have been minimized. Answer A is true, the sphere would be moving slowly at the end, but that doesn't really answer the question. Why would a slow final sphere velocity be better than a fast one? The "best" answer is C.

Now I'm beginning to wonder if the AAMC meant to compare a drop of 10m with an inclined plane of height 10m (and length > 10m)? That certainly is not clear from the question stem nor the passage itself.
 
Now I'm beginning to wonder if the AAMC meant to compare a drop of 10m with an inclined plane of height 10m (and length > 10m)? That certainly is not clear from the question stem nor the passage itself.

I'm almost positive that is what they meant by a 10m inclined plane.
 
Thanks for your replies MT and Axon. If in fact it was a misprint, which I also really think it was, this disappoints and worries me. Every question counts, and students simply can't afford to be lose points because of AAMC's mistakes, the MCAT is already very competitive as it is. I'm sure it's rare, but it really bugs me to know that they don't go the extra mile to make sure that there are absolutely no mistakes on the exam.
 
For BS #137:

Are we supposed to know that Cytochalasins are drugs that inhibit the growth of microfilaments? Or is there another way of getting the answer right?

Also for #140:

How are we supposed to know the answer is A? - That exocytosis of viral particles is proportional to microfilament polymerization.
 
137: The last paragraph of the passage states that the cytochalasins cap the + end of a microfilament, preventing additions there. It's hard to grow when the addition side is blocked.

140: I approached this one backwards. B and C are wrong because they would imply the opposite effect. D is weak because it didn't really address the release of the particles. Then I looked at A. I thought it sounded reasonable. More polymerization, more ability to generate force to push stuff out of the cell. I guess I answered this one with more of testing skill than biology skill. When they give you (OK, bad, bad, weak) you go with OK and move on. Maybe mark it for later if your confidence is low.
 
Passage III (red tide passage)
# 70: Why wouldn't D Work? The answer key says that D would mean that antidote would have to be applied and there could still be damage before antidote is applied. Well, can't one argue that for choice C, that would still be damage regardless of how it is slow/rare/easy to identify?
 
Paranoided my way to so many stupid answers on this test... kept looking for the tricks and got tripped up :(. Also have to work on my timing, finished too quickly on all of them and didnt go back enough.

Also, looking at it now I'm seeing that the concepts I didnt feel comfortable on I actually did better on because I didnt sit and confuse myself by trying to catch the 'trick' to the obviously hard question, made things more complex than they had to be. Also, humanities passages suck. Same problems though :/
 
Could someone provide a better explanation for this problem? I just can't grasp AAMC's explanation. Thanks.

Item 48

Solution
A battery in a circuit has an electromotive force given by and an internal resistance of r. The battery provides a current i to the circuit. What is the terminal voltage of the battery?

A g

B g - ir
The terminal voltage is the voltage provided to the external components of the circuit. The battery voltage ¼ will be reduced by the voltage required to overcome the internal resistance, so V = ¼ - ir. Thus, B is the best answer.

C g + ir

D g+ i2r
 
Could someone provide a better explanation for this problem? I just can't grasp AAMC's explanation. Thanks.

Item 48

Solution
A battery in a circuit has an electromotive force given by and an internal resistance of r. The battery provides a current i to the circuit. What is the terminal voltage of the battery?

A g

B g - ir
The terminal voltage is the voltage provided to the external components of the circuit. The battery voltage ¼ will be reduced by the voltage required to overcome the internal resistance, so V = ¼ - ir. Thus, B is the best answer.

C g + ir

D g+ i2r

unsure where "g" is coming from but I understand the question.

The internal resistance of a battery is a "flaw" that basically every battery has. (Not really a flaw, it's mostly just aging of the battery that causes the terminal voltage drop. Temperature can change it too but I'm not too sure how).

The battery itself has an impedance, before the net voltage can get to the rest of the circuit. The battery is not 100% efficient.

Therefore, the "terminal voltage" (ie- the NET voltage experienced by the rest of the circuit) will be less than than a 100% efficient/ideal.

The voltage could not be increased (or you would have an infinite energy source), only decreased. Thus B is correct.
 
unsure where "g" is coming from but I understand the question.

The internal resistance of a battery is a "flaw" that basically every battery has. (Not really a flaw, it's mostly just aging of the battery that causes the terminal voltage drop. Temperature can change it too but I'm not too sure how).

The battery itself has an impedance, before the net voltage can get to the rest of the circuit. The battery is not 100% efficient.

Therefore, the "terminal voltage" (ie- the NET voltage experienced by the rest of the circuit) will be less than than a 100% efficient/ideal.

The voltage could not be increased (or you would have an infinite energy source), only decreased. Thus B is correct.

Thanks. Never heard of the term "terminal voltage" before but thank you for the great explanation. I only put the g in there because it's the closest alphabetical letter to looking like that greek symbol in the question.
 
Passage III (red tide passage)
# 70: Why wouldn't D Work? The answer key says that D would mean that antidote would have to be applied and there could still be damage before antidote is applied. Well, can't one argue that for choice C, that would still be damage regardless of how it is slow/rare/easy to identify?

yes, choice C does not eliminate the fact that there are still dangers associated with red tides, even if they were less frequent, but the question was asking which choice would suggest that the author is EXAGGERATING about the dangers, not which choice would reduce/alleviate the damage caused by red tides. In the passage, the author mentioned that red tides are becoming more frequent, are hard to identify (the water doesn't always change color), and spread really fast. Choice C counters all 3 of these points made by the author. hope that makes sense!


Question: Item 87 (last question from the behaviorism passage in VR)
I can't understand the explanation for the right answer choice for the life of me... I didn't really understand what the passage was trying to say about functionalism to be honest. Can anyone explain to me why A is right??
 
Hey guys I don't really have a question on this question per se but rather on the statement bolded. From my understanding of the question, blood flow is being restricted to the kidneys and so we would expect an increase in the GFR. However, the statement says, "The reduced flow of blood through the renal arteries due to the clamps would cause a decrease in glomerular blood pressure." I guess my question is, how would this cause a decrease in the glomerular blood pressure? Shouldn't there be an increase since blood flow is restricted to the kidneys? Thanks everyone.

If restriction of blood flow to the kidneys (by placing clamps on the renal arteries) resulted in an immediate but small increase in blood pressure, followed by the gradual development of severe hypertension, which hypothesis would these results best support?

A Hypothesis A, because the clamps increased the vascular resistance to blood flow

B Hypothesis A, because the clamps caused the kidneys to receive less blood

C Hypothesis B, because the kidneys were responding to decreased glomerular blood pressure
The reduced flow of blood through the renal arteries due to the clamps would cause a decrease in glomerular blood pressure. The kidneys respond to this drop in pressure by activating the renin–angiotensin system of hormones. This increases the amount of sodium and water that is reabsorbed by the kidneys, therefore increasing blood volume and pressure. Thus, C is the best answer.

D Hypothesis B, because the volume of body fluids was probably decreasing
 
Hey guys I don't really have a question on this question per se but rather on the statement bolded. From my understanding of the question, blood flow is being restricted to the kidneys and so we would expect an increase in the GFR. However, the statement says, "The reduced flow of blood through the renal arteries due to the clamps would cause a decrease in glomerular blood pressure." I guess my question is, how would this cause a decrease in the glomerular blood pressure? Shouldn't there be an increase since blood flow is restricted to the kidneys? Thanks everyone.

If restriction of blood flow to the kidneys (by placing clamps on the renal arteries) resulted in an immediate but small increase in blood pressure, followed by the gradual development of severe hypertension, which hypothesis would these results best support?

A Hypothesis A, because the clamps increased the vascular resistance to blood flow

B Hypothesis A, because the clamps caused the kidneys to receive less blood

C Hypothesis B, because the kidneys were responding to decreased glomerular blood pressure
The reduced flow of blood through the renal arteries due to the clamps would cause a decrease in glomerular blood pressure. The kidneys respond to this drop in pressure by activating the renin&#8211;angiotensin system of hormones. This increases the amount of sodium and water that is reabsorbed by the kidneys, therefore increasing blood volume and pressure. Thus, C is the best answer.

D Hypothesis B, because the volume of body fluids was probably decreasing

Well think of it this way, when you clamp down on a hose at some point, no water comes out the other end right? Because you're blocking the water, no water pressure.
 
3 quick ps questions:

43. When aqueous solutions of the various anions and cations were mixed, precipitates formed because?

C. the solubilities of the cations were decreased by the other cations
D. The solubility product of a compound was exceeded.

Is C not correct mainly because they never specifiy if the cations are identical (ie common ion effect)? This makes D the best answer most likely?

40. According to the information in the passage, the gas evolves is:

C. carbon dioxide

-I saw some of the other explanations in the thread but I still dont entirely understand this question.

27. Which of the following items of information would NOT help in predicting the results shown in figure 1?

A. Number of air molecules inside the balloon
B. Thermal conductivity of the rubber
C. The variation with depth in the speed of the balloon
D. The total mass of the water in the tank

Answer: D I understand how A,B are important but how is C important in predicting the results in this question? I thought D would play a role in pressure.
 
also,

#117

In the patient described in the passage, the likely genetic basis of the increased levels of uric acid is a mutation:

A. Affecting the allosteric site of the PRPP synthetase
B. Affecting the active site of the PRPP synthetase

The explanation mentioned something that I wasnt sure about. The fact that the "active site" was not being affected becuase normal levels of the product were acheived in vitro. However, that made them see that the allosteric site was most likely the cause. Whats the difference between the allosteric site and the active site in this problem?
 
also,

#117

In the patient described in the passage, the likely genetic basis of the increased levels of uric acid is a mutation:

A. Affecting the allosteric site of the PRPP synthetase
B. Affecting the active site of the PRPP synthetase

The explanation mentioned something that I wasnt sure about. The fact that the "active site" was not being affected becuase normal levels of the product were acheived in vitro. However, that made them see that the allosteric site was most likely the cause. Whats the difference between the allosteric site and the active site in this problem?

An allosteric site on an enzyme is for regulating its activity through the binding of inhibitor/activator molecules. On the other hand, an activation site is where the enzyme interacts with its substrates and is separate from the allosteric site. So in this case, there's most likely a molecule in the body that acts as an activator to the mutated enzyme (when it binds to allosteric site, the enzyme activity increases), so its activity in the body is unusually high. Since the activation site is not affected and the activator molecule doesn't exist in vitro, the enzyme's activity is normal in vitro.
Hope that makes sense!
 
lNaxo.png


This question really really baffles me, cause the first thing the passage does is "make a distinction between literature and literary art". But AAMC is telling me that they're one and the same? WTF? The author is always careful to say "literary study" or "literary scholarship", and never uses the word "literature" as a substitute for the study of the literature. The Kaplan explanation is also bad; it says "C" is wrong because the author never says anything about artistic pursuit. But in the second sentence he states "The two are distinct activates: [Literature] is creative, an art..."

So bothered by this.

Rofl so I want to share the dumbest error I've mad thus far in MCAT practice. Question 118, in BS regarding which animal excretes nitrogen primarily as nitric acid, for some reason I read answer choice C, carnivorous shark, as carnivorous SNAKE. I remember that reptiles excrete uric acid, and I wasn't sure birds do. And I even checked my answer a few times and looked at it again after I realized I got it wrong, but only now while carefully going through the test did I realize it actually said shark. SMH
 
Last edited:
LOTS of irks with AAMC8...but still completely stoked on my score!!

If you guys could help me out I'd appreciate it.

PS: Question 50: I thought frequency was the 1 thing that couldn't change in a wave.

Is the "imperfect" reflector an indication that everything changes except the speed? I'm not sure what an "imperfect" reflector is. The explanation doesn't do it for me...

VR: Question 61:
I chose answer "A." The explanation says the answer is D, which I was going to put, however, the passage NEVER mentions Red Riding Hood, and choice D FORCES you to assume that Red Riding Hood has enduring popularity (which it does, but my VR muscle has been toned to never pick answer that force you to assume things), so why would they all of sudden make you assume this?

Question 80:
I chose answer "D." I had it down to C and D and picked C but reviewing I put D. The author obviously understands the historical significance, but he refers to classic approach as being around "ever since", thus up until this day. IE- it has endured through history and will continue to endure, so he accepts that it will forever be dominated by religion and politics. Then again "religion" shouldve been replaced with spirituality for this to be more right...

Question 86: I put "C"...why would anyone replace the original rope if they were influenced by beliefs that had nothing to do with the original rope...

Question 88: I answer choice "B" based on the explicitly stated "Learning activities are more limited to their usefulness in beginners..."

BS: Question 121:
I put choice "B" and the answer is "D". The most plausible explanation is for you to assume that one of the fatty chains was unsaturated? I know fatty chains can be unsaturated, but this answer is better than a glycerol losing a proton to form a fatty acid salt? Then again, it's really not a fatty acid salt, just a glycerol soap...ehh.

Question 135: At which concentration will the + grow faster than the - ... I put "D" at any concentration, because it states in the passage "During microfilament growth, the + end grows faster than the - end." and the fact that growth (which is a rate) of "+" has a greater slope than "-" at all concentrations...


Thanks in advance to anyone who actually spends time to read this and help me out.
 
3 quick ps questions:

43.Is C not correct mainly because they never specifiy if the cations are identical (ie common ion effect)? This makes D the best answer most likely?

40. According to the information in the passage, the gas evolves is:

-I saw some of the other explanations in the thread but I still dont entirely understand this question.

27. Which of the following items of information would NOT help in predicting the results shown in figure 1?

A. Number of air molecules inside the balloon
B. Thermal conductivity of the rubber
C. The variation with depth in the speed of the balloon
D. The total mass of the water in the tank

Answer: D I understand how A,B are important but how is C important in predicting the results in this question? I thought D would play a role in pressure.


43: Yes.

40: I got this correct just because CO would probably not be emitted from the mixture rapidly, and the fact that CO2 is often a product of reactions with carbonate. Not entirely sure why sulfur couldn't be evolved, though.

I'm not sure of the exact reasoning of the AAMC either. HCl reacts with CO3 2- to produce CO2 and ... NiCl2, H2O and generates the H+ from the original HCl.

27: Choice "D" does play some role in pressure, but it's not the best answer in predicting the temperature and depth of the balloon. It doesn't relate anything to the energy being asked. The mass of water is basically arbitrary. If you had the ocean to work with, the results in figure 1 would look the same because the balloon would go straight up. If you had a long pipe on its end you would still get the same results. The total mass of water doesn't really matter.

I can see where you're coming from but based on the information you're given, D is the best answer.
 
Question 135: At which concentration will the + grow faster than the - ... I put "D" at any concentration, because it states in the passage "During microfilament growth, the + end grows faster than the - end." and the fact that growth (which is a rate) of "+" has a greater slope than "-" at all concentrations...

That was definitely a problematic question...I chose the same answer as you did because I assumed that growth was a rate too (instead of one end getting longer). but I guess they meant "growth" as one strand actually having a positive rate of growth. =/
 
PS #12: According to the passage, at resonance, B2 rotates an H nucleus through an angle of?

Answer: 180 degrees

Why? Why isn't it 90 degrees?

The passage says that B1 (stationary) and B2 (rotating) are always perpendicular to each other.

The passage also says that at resonance, B1 is antiparallel (180 degrees) to the H nucleus.

Doesn't it follow then, at resonance, that B2 would be 90 degrees to the H nucleus?
 
Ok guys for

BS 102 - I noticed that the melting point of 2E was very close to the melting point of 2-furaldehyde, which is the precise reason I selected it as NOT the more stable product.

The answer assumes that the semicarbazone will have the same melting point as its precursor, but I figured that the reason the melting point was a little lower was because of impurities (semicarbazone).

Thus, if there was 2-furaldehyde in the flask and the semicarbazone of cyclohexanone, then the melting point would be less than that of pure 2-furaldehyde.

Or, are you supposed to know that the semicarbazone and its precursor will have the same melting point?
 
PS #12: According to the passage, at resonance, B2 rotates an H nucleus through an angle of?

Answer: 180 degrees

Why? Why isn't it 90 degrees?

The passage says that B1 (stationary) and B2 (rotating) are always perpendicular to each other.

The passage also says that at resonance, B1 is antiparallel (180 degrees) to the H nucleus.

Doesn't it follow then, at resonance, that B2 would be 90 degrees to the H nucleus?

I said the same thing, but 270 degrees and 90 degrees are the same thing, so there can't be 2 answers. 45 degrees doesn't make sense so you're left with 180 degrees.

Still don't know how 180 degrees is justified other than the fact that none of the other answers were correct. It's like saying I know these are wrong, and don't know about this, but I'm picking it anyways.
 
hmm, ghey. maybe they meant to write B1 but wrote B2 by accident. thanks for giving me your POE skills though.
 
But it is rotated from being parallel with B1 (stated) to antiparallel with B1. This is 180&#730;.
 
13 PS, 12 VR, 13 BS Here. Thayt verbal passage on behaviorism was something else...

This has been asked before but I need a major bump. EK did not touch on this AT ALL.

"Which semicarbazone is the product of thermodynamic control?"

Answer:

C - Furzaldehyde's because it is produced under equilibirium conditions and is more stable than the other product."

Ok, I did NOT COMPUTE THIS PASSAGE AT ALL.

A. What is a semicarbazone?

B. What are the definitions of kinetic/thermo controls?

C. Why is C the answer?

Thank you!
 
Yeah, I didn't know what the hell a semicarbazone was either.

Reactions of certain compounds can either form a thermodynamic product (the more stable product) or a kinetic product (the one with the lower activation energy).

The thermodynamic pathway has a higher activation energy, so at high temperatures it will predominate.

The kinetic pathway has a lower activation energy, so at lower temperatures it will predominate.
 
I understand how removal of parathyroid gland could cause hypocalcemia, and thus induce severe neural and muscular problems. However, in the explanation it states that hypocalcemia would cause increased neuromuscular excitability because of change in membrane potential. I don't understand this. Because the use of calcium is in transmission of neural impulses across synapses in both neural and muscular synapses. When Ca2+ enters the synapse, neurotransmitter is released. However, when there is deficiency in calcium, neural impulse would not be transmitted to another neuron or to a muscle. Hence, excitation should actually decrease, right? Hence, there should be less spasms....but the answer says quite the opposite!!!!!
:confused:
 
I understand how removal of parathyroid gland could cause hypocalcemia, and thus induce severe neural and muscular problems. However, in the explanation it states that hypocalcemia would cause increased neuromuscular excitability because of change in membrane potential. I don't understand this. Because the use of calcium is in transmission of neural impulses across synapses in both neural and muscular synapses. When Ca2+ enters the synapse, neurotransmitter is released. However, when there is deficiency in calcium, neural impulse would not be transmitted to another neuron or to a muscle. Hence, excitation should actually decrease, right? Hence, there should be less spasms....but the answer says quite the opposite!!!!!
:confused:

Nope, look at it this way: it's all about the gradient. Less calcium extracellularly means that the Ca 2+ gradient across the membrane is not very skewed at all. Hence, Neurotransmitters may even be released without Ca 2+ channels opening, because the gradient is so skewed in the wrong direction.

Hope that helps!
 
That's a different outlook. But it still would seem more logical to think that the amount of neural impulse sent decreases, even though according to this it should increase.
 
Can someone explain question 133 to me?

Which of the following conclusions about dewlap reflectance is supported by the passage?

A. lizard habitat is determined by dewlap reflectance for each species
b. high dewlap reflectance is most important in brightly lit habitats
c - wrong
d - wrong.

I put A, but the answer is B. I don't understand how you can jump to the conclusion stated by B. Sure the 3 species live in sunny areas, but how can you state that they're important because they live there? Causation doesn't mean correlation. Plus, I felt like A exactly stated what was in the passage: "There is a relationship between UV reflectance and habitat."

Can anyone clear this up?
 
Can someone explain question 133 to me?

Which of the following conclusions about dewlap reflectance is supported by the passage?

A. lizard habitat is determined by dewlap reflectance for each species
b. high dewlap reflectance is most important in brightly lit habitats
c - wrong
d - wrong.

I put A, but the answer is B. I don't understand how you can jump to the conclusion stated by B. Sure the 3 species live in sunny areas, but how can you state that they're important because they live there? Causation doesn't mean correlation. Plus, I felt like A exactly stated what was in the passage: "There is a relationship between UV reflectance and habitat."

Can anyone clear this up?

If it's an adaptation, it's important and offered a selective advantage.
 
Top